What is the Remainder of Polynomial Division in Z5[x] by x+3?

Click For Summary
The problem involves finding the remainder of the polynomial x^4 + 3x + 2 when divided by x + 3 in the ring Z5[x]. The initial calculation yielded a quotient of x^3 + 2x^2 + 4x + 1, with a remainder of 4. A detailed verification shows that multiplying the divisor by the quotient results in x^4 + 3x + 3 modulo 5, confirming that the correct remainder is indeed 4. The discussion highlights the importance of careful polynomial division in different rings, particularly in Z5. The conclusion affirms that the remainder of the division is correctly identified as 4.
sarah77
Messages
27
Reaction score
0

Homework Statement



Find the remainder of x^4 + 3x +2 after division by x+3 in Z5[x]

Homework Equations



my quotient after dividing was: x^3 + 2X^2 + 4x +1

The Attempt at a Solution



I found the remainder to be 4. If anyone has time, I believe I made a mistake somewhere and would like someone to attempt this division so I can check my work. If you have time please check it because I am not confident in dividing polynomials of different rings. Thank you
 
Physics news on Phys.org
> I found the remainder to be 4. If anyone has time, I believe I made a mistake somewhere and would like someone to attempt this division so I can check my work. If you have time please check it because I am not confident in dividing polynomials of different rings.

Since (x+3)(x^3 + 2x^2 + 4x + 1) = x^4 + 2x^3 + 4x^2 + x + 3x^3 + 6x^2 + 12x + 3 = x^4 + 5x^3 + 10x^2 + 13x + 3 = x^4 + 3x + 3 (mod 5), I deduce that the remainder is -1 = 4 (mod 5). What makes you think you (and therefore also I) made a mistake somewhere?
 
Question: A clock's minute hand has length 4 and its hour hand has length 3. What is the distance between the tips at the moment when it is increasing most rapidly?(Putnam Exam Question) Answer: Making assumption that both the hands moves at constant angular velocities, the answer is ## \sqrt{7} .## But don't you think this assumption is somewhat doubtful and wrong?

Similar threads

  • · Replies 3 ·
Replies
3
Views
2K
  • · Replies 5 ·
Replies
5
Views
2K
  • · Replies 18 ·
Replies
18
Views
5K
  • · Replies 4 ·
Replies
4
Views
3K
  • · Replies 4 ·
Replies
4
Views
1K
  • · Replies 8 ·
Replies
8
Views
2K
Replies
10
Views
2K
  • · Replies 1 ·
Replies
1
Views
1K
  • · Replies 14 ·
Replies
14
Views
2K
  • · Replies 11 ·
Replies
11
Views
2K